LSAT Explanation PT 41, S1, Q24: Ethicist: In general it is wrong

LSAT Question Stem

Which one of the following is an assumption required by the ethicist's argument? 

Logical Reasoning Question Type

This is a Necessary Assumption question. 

Correct Answer

The correct answer to this question is E. 

LSAT Question Complete Explanation

The argument in the passage can be summarized as follows: The ethicist acknowledges that it's generally wrong to use experimental medical treatments without a patient's consent due to their right to be fully informed. However, they argue that in emergency situations, some nonconsensual medical research should be allowed since it can lead to the discovery of the best treatment options. The structure of the argument consists of two premises and a conclusion. The first premise is about the patient's right to informed consent, the second premise is about the necessity of nonconsensual research in emergencies, and the conclusion is that nonconsensual medical research should be allowed in some cases.

To make the topic more relatable, consider this simplified example: Imagine you're at a restaurant and have a severe allergic reaction. The chef has an experimental dish that could potentially help you, but they don't have time to explain it to you and get your consent. In this case, the ethicist would argue that it's acceptable for the chef to serve you the dish without your consent due to the emergency situation.

An "Evaluate" question for this argument could be: "Do the potential benefits of nonconsensual medical research in emergencies outweigh the importance of a patient's right to informed consent?"

The question type for this problem is Necessary Assumption, which asks us to identify an assumption required by the ethicist's argument.

Answer Choice Explanations:

a) While doctors' knowledge in emergency situations might be relevant, it is not a necessary assumption for the ethicist's argument. The argument is focused on the balance between patients' rights and the benefits of medical research in emergencies.

b) This answer choice might seem tempting at first, but the assumptions underlying doctors' decisions to bypass consent are not directly relevant to the argument in favor of nonconsensual medical research. The focus is on the balance between patients' rights and research benefits.

c) This choice weakens the argument by limiting the application of the principle used by the ethicist. The argument doesn't require that nonconsensual research should only be allowed if it's highly likely to benefit the patient, just that it should be allowed in some cases.

d) This choice is not relevant to the conclusion, as it discusses patients' rights in cases where the best treatment option is unknown. The argument is about allowing nonconsensual research in emergencies, not about patients' rights in general.

e) This is the correct answer choice. It states that the ethicist must assume that the potential benefits of nonconsensual medical research in emergencies outweigh the importance of a patient's right to informed consent. This assumption is necessary for the argument to be valid. Using the Assumption Negation technique, if this assumption was negated (i.e., patients' right to informed consent is never outweighed by research benefits), the conclusion would be weakened, confirming that this is the necessary assumption.

Previous
Previous

LSAT Explanation PT 42, S2, Q6: Commentator: In many countries the influence

Next
Next

LSAT Explanation PT 40, S3, Q24: New evidence suggests that the collapse